Difference between revisions of "2023 AMC 8 Problems/Problem 16"
m |
m (Formatting) |
||
Line 3: | Line 3: | ||
The letters <math>\text{P}, \text{Q},</math> and <math>\text{R}</math> are entered into a <math>20\times20</math> table according to the pattern shown below. How many <math>\text{P}</math>s, <math>\text{Q}</math>s, and <math>\text{R}</math>s will appear in the completed table? | The letters <math>\text{P}, \text{Q},</math> and <math>\text{R}</math> are entered into a <math>20\times20</math> table according to the pattern shown below. How many <math>\text{P}</math>s, <math>\text{Q}</math>s, and <math>\text{R}</math>s will appear in the completed table? | ||
<asy> | <asy> | ||
− | |||
usepackage("mathdots"); | usepackage("mathdots"); | ||
size(5cm); | size(5cm); | ||
Line 72: | Line 71: | ||
<math>\textbf{(E)}~134\text{ Ps, }133\text{ Qs, }133\text{ Rs}</math> | <math>\textbf{(E)}~134\text{ Ps, }133\text{ Qs, }133\text{ Rs}</math> | ||
− | == Solution 1 ( | + | == Solution 1 (Finding a Pattern)== |
− | In our <math>5 \ | + | In our <math>5\times5</math> grid, there are <math>8,9</math> and <math>8</math> of the letters <math>\text{P}, \text{Q},</math> and <math>\text{R}</math>, respectively, and in a <math>2\times2</math> grid, there are <math>1,2</math> and <math>1</math> of the letters <math>\text{P}, \text{Q},</math> and <math>\text{R}</math>, respectively. We see that in both grids, there are <math>x, x+1,</math> and <math>x</math> of the <math>\text{P}, \text{Q},</math> and <math>\text{R}</math>, respectively. This is because in any <math>n\times n</math> grid with <math>n\equiv2\pmod3</math>, there are <math>x, x+1,</math> and <math>x</math> of the <math>\text{P}, \text{Q},</math> and <math>\text{R}</math>, respectively. We can see that the only answer choice which satisfies this condition is <math>\boxed{\textbf{(C)}~133\text{ Ps, }134\text{ Qs, }133\text{ Rs}}.</math> |
− | + | ~CoOlPoTaToEs, apex304, SohumUttamchandani, wuwang2002, TaeKim, Cxrupptedpat, Nivaar | |
− | |||
− | ~apex304, SohumUttamchandani, wuwang2002, TaeKim, Cxrupptedpat, Nivaar | ||
== Solution 2 == | == Solution 2 == | ||
− | + | Since <math>20\equiv2\pmod3</math> and <math>\text{Q}</math> is in the 2nd diagonal, it is also in the 20th diagonal, and so we find that there are <math>2(2 + 5 + 8 + 11 + 14 + 17) + 20 = 134 \text{Qs}</math>. Since all the <math>\text{P}</math>'s and <math>\text{R}</math>'s are symmetric, the answer is <math>\boxed{\textbf{(C)}~133\text{ Ps, }134\text{ Qs, }133\text{ Rs}}.</math> | |
~[[User:ILoveMath31415926535|ILoveMath31415926535]] | ~[[User:ILoveMath31415926535|ILoveMath31415926535]] | ||
Line 92: | Line 89: | ||
~mathboy100 | ~mathboy100 | ||
− | == Solution 4 (Brute | + | == Solution 4 (Brute Force) == |
− | |||
<asy> | <asy> | ||
− | |||
usepackage("mathdots"); | usepackage("mathdots"); | ||
size(16.666cm); | size(16.666cm); | ||
Line 162: | Line 157: | ||
drawDiagonal("R", (19.5,19.5)); | drawDiagonal("R", (19.5,19.5)); | ||
</asy> | </asy> | ||
− | <b>This solution is extremely time-consuming and error-prone. | + | |
+ | From the full diagram, the answer is <math>\boxed{\textbf{(C)}~133\text{ Ps, }134\text{ Qs, }133\text{ Rs}}.</math> | ||
+ | <b>This solution is extremely time-consuming and error-prone. Do not try it in a real competition unless you have no other options.</b> | ||
~MRENTHUSIASM | ~MRENTHUSIASM | ||
Line 183: | Line 180: | ||
== Solution 6 (Modular Arithmetic)== | == Solution 6 (Modular Arithmetic)== | ||
− | + | If a letter is in a horizontal position <math>k</math>, then that same letter will appear in position <math>k+3m</math>, for a positive integer <math>m</math>. In other words, all positions congruent to <math>k</math> modulo <math>3</math> will have the same letter as <math>p</math>. | |
− | |||
− | |||
Since <math>p</math> is in position <math>1</math>, <math>p</math> will be in every position congruent to <math>1 \pmod 3</math>. There are <math>7</math> numbers less than or equal to <math>20</math> that satisfy this restraint. There are also <math>7</math> numbers less than or equal to <math>2</math> that are congruent to <math>2 \pmod 3</math>, but only <math>6</math> that are multiples of <math>3</math>. | Since <math>p</math> is in position <math>1</math>, <math>p</math> will be in every position congruent to <math>1 \pmod 3</math>. There are <math>7</math> numbers less than or equal to <math>20</math> that satisfy this restraint. There are also <math>7</math> numbers less than or equal to <math>2</math> that are congruent to <math>2 \pmod 3</math>, but only <math>6</math> that are multiples of <math>3</math>. | ||
− | In <math>p</math>'s case, it will appear <math>7</math> times in row one, only <math>6</math> in row <math>2</math> (as its first | + | In <math>p</math>'s case, it will appear <math>7</math> times in row one, only <math>6</math> in row <math>2</math> (as its first appearance is in position <math>3</math>), and <math>7</math> in row <math>3</math>. So in the first <math>3</math> rows, <math>P</math> appears <math>20</math> times. |
Therefore, in the first <math>18</math> rows, <math>P</math> appears <math>20\cdot 6 = 120</math> times. Row <math>19</math> looks identical to row <math>1</math>, as <math>19\equiv 1\pmod 3</math>, so <math>P</math> appears in row <math>19</math> <math>7</math> times. It follows that <math>P</math> appears in row <math>20</math> <math>6</math> times. There are <math>134 P</math>s. | Therefore, in the first <math>18</math> rows, <math>P</math> appears <math>20\cdot 6 = 120</math> times. Row <math>19</math> looks identical to row <math>1</math>, as <math>19\equiv 1\pmod 3</math>, so <math>P</math> appears in row <math>19</math> <math>7</math> times. It follows that <math>P</math> appears in row <math>20</math> <math>6</math> times. There are <math>134 P</math>s. | ||
Line 209: | Line 204: | ||
~andyluo | ~andyluo | ||
− | ==Solution 8 | + | ==Solution 8 (Intuitive, Fastest)== |
− | |||
− | |||
− | |||
− | |||
− | |||
− | |||
− | |||
− | |||
− | |||
− | |||
− | |||
− | |||
− | |||
− | |||
When we find the letters at the corners, we see that there are 2 other corners with Q's and 1 with R. There are 2 corners with Q and 1 with P and R. Therefore, it makes sense that there should be more Q's than other letters. Only <math>\boxed{\textbf{(C)}~133\text{ Ps, }134\text{ Qs, }133\text{ Rs}}</math> has more Q's than other letters. | When we find the letters at the corners, we see that there are 2 other corners with Q's and 1 with R. There are 2 corners with Q and 1 with P and R. Therefore, it makes sense that there should be more Q's than other letters. Only <math>\boxed{\textbf{(C)}~133\text{ Ps, }134\text{ Qs, }133\text{ Rs}}</math> has more Q's than other letters. | ||
− | > | + | <b>Only do it this way if there are 30 seconds left on the clock, as it may not always work!</b> |
− | ==Solution | + | ==Solution 9 (Think and Label)== |
− | Since there are <math>3</math> letters, there must be a multiple of 3 for the height of the array for the amount of letters to be the same. However, there is one less (<math>21-1=20</math>) so there will be one less letter in each column compared to the other two. Looking at the diagram, the first column starts with a <math>P</math> then a <math>Q</math>. Since it will miss the third letter, <math>R</math>, we write this as <math>-R</math>. We do this process for the remaining 20 columns, and applying the same logic if the first three are <math>-R</math>, <math>-P</math> and <math>-Q</math>, there will be one less <math>-Q</math> | + | Since there are <math>3</math> letters, there must be a multiple of 3 for the height of the array for the amount of letters to be the same. However, there is one less (<math>21-1=20</math>) so there will be one less letter in each column compared to the other two. Looking at the diagram, the first column starts with a <math>P</math> then a <math>Q</math>. Since it will miss the third letter, <math>R</math>, we write this as <math>-R</math>. We do this process for the remaining 20 columns, and applying the same logic if the first three are <math>-R</math>, <math>-P</math> and <math>-Q</math>, there will be one less <math>-Q</math> meaning that there will be an extra <math>Q</math>. The only answer that has in extra <math>Q</math> is <math>\textbf{(C)}~133\text{ Ps, }134\text{ Qs, }133\text{ Rs}</math> |
− | <math>\textbf{(C)}~133\text{ Ps, }134\text{ Qs, }133\text{ Rs}</math> | ||
~Blue_Kite | ~Blue_Kite | ||
− | ==Video Solution by Math-X== | + | == Video Solution 1 by Math-X == |
+ | |||
https://youtu.be/Ku_c1YHnLt0?si=Xxe3CXtrXQYGSYik&t=3081 | https://youtu.be/Ku_c1YHnLt0?si=Xxe3CXtrXQYGSYik&t=3081 | ||
− | + | == Video Solution 2 by CoolMathProblems == | |
− | |||
https://youtu.be/_huZfhiCBN8 | https://youtu.be/_huZfhiCBN8 | ||
− | ==Video Solution | + | == Video Solution 3 by hnsacademy == |
+ | |||
https://youtu.be/zntZrtsnyxc?si=nM5eWOwNU6HRdleZ&t=1453 | https://youtu.be/zntZrtsnyxc?si=nM5eWOwNU6HRdleZ&t=1453 | ||
− | |||
− | ==Video Solution | + | == Video Solution 4 == |
+ | |||
https://youtu.be/3DTwjLe0Pw0 | https://youtu.be/3DTwjLe0Pw0 | ||
~Education, the Study of Everything | ~Education, the Study of Everything | ||
− | ==Video Solution | + | == Video Solution 5 == |
+ | |||
https://youtu.be/1tnMR0lNEFY | https://youtu.be/1tnMR0lNEFY | ||
− | ~ | + | ~[//starleague.us Star League] |
+ | |||
+ | == Video Solution 6 by OmegaLearn (Using Cyclic Patterns) == | ||
− | |||
https://youtu.be/83FnFhe4QgQ | https://youtu.be/83FnFhe4QgQ | ||
− | ==Video Solution by Magic Square== | + | == Video Solution 7 by Magic Square == |
+ | |||
https://youtu.be/-N46BeEKaCQ?t=3990 | https://youtu.be/-N46BeEKaCQ?t=3990 | ||
− | ==Video Solution by Interstigation== | + | |
+ | == Video Solution 8 by Interstigation == | ||
+ | |||
https://youtu.be/DBqko2xATxs&t=1845 | https://youtu.be/DBqko2xATxs&t=1845 | ||
− | ==Video Solution by WhyMath== | + | == Video Solution 9 by WhyMath == |
+ | |||
https://youtu.be/iMhqlz0-ce0 | https://youtu.be/iMhqlz0-ce0 | ||
− | + | == Video Solution 10 == | |
− | |||
https://www.youtube.com/watch?v=jtHe6yLBz-4&t=20s | https://www.youtube.com/watch?v=jtHe6yLBz-4&t=20s | ||
− | + | == Video Solution 11 by Dr. David == | |
− | |||
https://youtu.be/yeXuFQHYU7k | https://youtu.be/yeXuFQHYU7k | ||
− | ==See Also== | + | ==See Also== |
+ | |||
{{AMC8 box|year=2023|num-b=15|num-a=17}} | {{AMC8 box|year=2023|num-b=15|num-a=17}} | ||
{{MAA Notice}} | {{MAA Notice}} |
Revision as of 10:13, 19 January 2025
Contents
- 1 Problem
- 2 Solution 1 (Finding a Pattern)
- 3 Solution 2
- 4 Solution 3
- 5 Solution 4 (Brute Force)
- 6 Solution 5
- 7 Solution 6 (Modular Arithmetic)
- 8 Solution 7 (Answer Choices, Fast)
- 9 Solution 8 (Intuitive, Fastest)
- 10 Solution 9 (Think and Label)
- 11 Video Solution 1 by Math-X
- 12 Video Solution 2 by CoolMathProblems
- 13 Video Solution 3 by hnsacademy
- 14 Video Solution 4
- 15 Video Solution 5
- 16 Video Solution 6 by OmegaLearn (Using Cyclic Patterns)
- 17 Video Solution 7 by Magic Square
- 18 Video Solution 8 by Interstigation
- 19 Video Solution 9 by WhyMath
- 20 Video Solution 10
- 21 Video Solution 11 by Dr. David
- 22 See Also
Problem
The letters and are entered into a table according to the pattern shown below. How many s, s, and s will appear in the completed table?
Solution 1 (Finding a Pattern)
In our grid, there are and of the letters and , respectively, and in a grid, there are and of the letters and , respectively. We see that in both grids, there are and of the and , respectively. This is because in any grid with , there are and of the and , respectively. We can see that the only answer choice which satisfies this condition is
~CoOlPoTaToEs, apex304, SohumUttamchandani, wuwang2002, TaeKim, Cxrupptedpat, Nivaar
Solution 2
Since and is in the 2nd diagonal, it is also in the 20th diagonal, and so we find that there are . Since all the 's and 's are symmetric, the answer is
Solution 3
Notice that rows and are the same, for any Additionally, rows and collectively contain the same number of s, s, and s, because the letters are just substituted for one another. Therefore, the number of s, s, and s in the first rows is . The first row has s, s, and s, and the second row has s, s, and s. Adding these up, we obtain .
~mathboy100
Solution 4 (Brute Force)
From the full diagram, the answer is This solution is extremely time-consuming and error-prone. Do not try it in a real competition unless you have no other options.
~MRENTHUSIASM
Solution 5
This solution refers to the full diagram in Solution 4.
Note the diagonals are symmetric. The and diagonals are not symmetric, but are reflections of each other about the diagonals:
- The upper diagonal of length is surrounded by a diagonal of length and an diagonal of length
- The lower diagonal of length is surrounded by a diagonal of length and an diagonal of length
When looking at a pair of diagonals of the same length there is a total of s and s next to these diagonals.
The main diagonal of s has s and s next to it. Thus, the total is s, s, s. Therefore, the answer is
~ERMSCoach
Solution 6 (Modular Arithmetic)
If a letter is in a horizontal position , then that same letter will appear in position , for a positive integer . In other words, all positions congruent to modulo will have the same letter as .
Since is in position , will be in every position congruent to . There are numbers less than or equal to that satisfy this restraint. There are also numbers less than or equal to that are congruent to , but only that are multiples of .
In 's case, it will appear times in row one, only in row (as its first appearance is in position ), and in row . So in the first rows, appears times.
Therefore, in the first rows, appears times. Row looks identical to row , as , so appears in row times. It follows that appears in row times. There are s.
Counting s is nearly identical, but begins in position . In the first rows, there are an identical amount of s too, namely . However, by a similar argument to , appears times in the last two rows; because row is the same as row , appears in position , and thus times.
Therefore, there are s and . We could go through a similar argument for , or note that the only answer choice with these two options is
-Benedict T (countmath1)
Solution 7 (Answer Choices, Fast)
We can first observe that shows on diagonals increasing or decreasing by
It starts at and increases in the form . Using our answer choices, and are the only fits.
is like this as well, increasing This means has to be in the form of Testing this out leads us with
~andyluo
Solution 8 (Intuitive, Fastest)
When we find the letters at the corners, we see that there are 2 other corners with Q's and 1 with R. There are 2 corners with Q and 1 with P and R. Therefore, it makes sense that there should be more Q's than other letters. Only has more Q's than other letters.
Only do it this way if there are 30 seconds left on the clock, as it may not always work!
Solution 9 (Think and Label)
Since there are letters, there must be a multiple of 3 for the height of the array for the amount of letters to be the same. However, there is one less () so there will be one less letter in each column compared to the other two. Looking at the diagram, the first column starts with a then a . Since it will miss the third letter, , we write this as . We do this process for the remaining 20 columns, and applying the same logic if the first three are , and , there will be one less meaning that there will be an extra . The only answer that has in extra is
~Blue_Kite
Video Solution 1 by Math-X
https://youtu.be/Ku_c1YHnLt0?si=Xxe3CXtrXQYGSYik&t=3081
Video Solution 2 by CoolMathProblems
Video Solution 3 by hnsacademy
https://youtu.be/zntZrtsnyxc?si=nM5eWOwNU6HRdleZ&t=1453
Video Solution 4
~Education, the Study of Everything
Video Solution 5
Video Solution 6 by OmegaLearn (Using Cyclic Patterns)
Video Solution 7 by Magic Square
https://youtu.be/-N46BeEKaCQ?t=3990
Video Solution 8 by Interstigation
https://youtu.be/DBqko2xATxs&t=1845
Video Solution 9 by WhyMath
Video Solution 10
https://www.youtube.com/watch?v=jtHe6yLBz-4&t=20s
Video Solution 11 by Dr. David
See Also
2023 AMC 8 (Problems • Answer Key • Resources) | ||
Preceded by Problem 15 |
Followed by Problem 17 | |
1 • 2 • 3 • 4 • 5 • 6 • 7 • 8 • 9 • 10 • 11 • 12 • 13 • 14 • 15 • 16 • 17 • 18 • 19 • 20 • 21 • 22 • 23 • 24 • 25 | ||
All AJHSME/AMC 8 Problems and Solutions |
The problems on this page are copyrighted by the Mathematical Association of America's American Mathematics Competitions.